about choice A
why is A wrong? someone please explain.
Dina on November 23 at 01:28AM
  • June 1999 LSAT
  • SEC2
  • Q9
1
Reply
about this question?
could someone clarify and explain this question please? thank you.
Dina on August 2 at 10:34AM
  • June 1999 LSAT
  • SEC2
  • Q23
2
Replies
help please
I don't understand why A is incorrect. I was between B and A. Can someone please explain why? Tha...
Abigail-Okereke on October 30, 2022
  • June 1999 LSAT
  • SEC2
  • Q9
1
Reply
how is A possible
i dont see L next to N N can only be in 6 or 2 With N at 6 L is in either 2 or 4 With N at 2 L...
medasmx@protonmail.com on January 22, 2022
  • June 1999 LSAT
  • SEC2
  • Q5
1
Reply
How is C the right answer?
In my scenario for this question C is a possible, it could be true. If you place them like N>L>P>...
cass on September 20, 2021
  • June 1999 LSAT
  • SEC2
  • Q5
1
Reply
Can I see a set up for this question
I was a little lost on how to set up this question, mainly because I'm not great at games that th...
Terius-Madrigal on July 16, 2021
  • June 1999 LSAT
  • SEC2
  • Q6
1
Reply
missing a rule?
Please explain this answer. It seems R, S, T can be swapped amongst GHLP
JasonB on June 12, 2021
  • June 1999 LSAT
  • SEC2
  • Q7
2
Replies
Video explanation
The video explanation for this question is not working. Can someone please fix it?
Joelle18 on April 17, 2021
  • June 1999 LSAT
  • SEC2
  • Q15
1
Reply
Why are there no videos for explaining the ques...
Why are there no videos for explaining the questions?
Selma-Ismail on January 4, 2021
  • June 1999 LSAT
  • SEC2
  • Q13
1
Reply
Typo reporting.
I am pretty much sure that this answer is a typo. Your team may need to check out and correct thi...
Batman on October 10, 2020
  • June 1999 LSAT
  • SEC2
  • Q9
3
Replies
This stem is missing information
I have spent an hour doing this game over and over and trying to understand why C is unacceptable...
jordanbirnholtz on June 6, 2020
  • June 1999 LSAT
  • SEC2
  • Q10
2
Replies
What is the correct answer?
"10 more actual official lsat" book gives as a correct answer "c" to question 5. Why not "E". Mos...
annag on October 4, 2014
  • June 1999 LSAT
  • SEC2
  • Q5
1
Reply